LSAT and Law School Admissions Forum

Get expert LSAT preparation and law school admissions advice from PowerScore Test Preparation.

User avatar
 Dave Killoran
PowerScore Staff
  • PowerScore Staff
  • Posts: 5852
  • Joined: Mar 25, 2011
|
#59784
Complete Question Explanation
(The complete setup for this game can be found here: lsat/viewtopic.php?t=26564)

The correct answer choice is (A)

The inclusion of “Except” in the question stem turns the meaning of the question “upside down,” and so the four incorrect answer choices have the characteristic of Could Be True, and the one correct answer choice has the characteristic of Cannot Be True. In this way, the question can be analyzed as a simple Cannot Be True question, and in doing so we have negated the confusing effect of the word “except.”

By adding the “if” statement in the question stem to our original rules, we arrive at the following chain sequence:


..... ..... ..... ..... ..... O :longline: M :longline: T


Once again, this is a Local question and the best approach is to make a mini-diagram next to the question, again accounting for the two basic templates created by the MP split-block:

pt26_s98_g2_q12.png

In the first template, P is delivered last and M is delivered fifth. According to the third rule, T must then be delivered sixth. The remaining four variables—L, N, O, and S—must then be delivered in the first four positions, in accordance with the remaining rules. The parentheses around L, N, O, and S are an efficient notation that indicates the enclosed variables are to be placed in the consecutive spaces in some order.

In the second template, P is delivered first and M is delivered third. According to the condition in the question stem, O must then be delivered second. And, because O is delivered second, L must then be delivered fourth as dictated by the fourth rule. The remaining three variables—N, S, and T—must then be delivered in the final three spaces in some order.

Since the question stem specifically focuses on the fifth package delivered, we can use our mini-diagram to quickly deduce which packages can be delivered fifth and then use that information to eliminate answer choices. From the first template it is proven that package M can be delivered fifth, and thus answer choice (B) can be eliminated. From the second template, it can be determined that packages N, S, and T can be delivered fifth, and this information eliminates answer choices (C), (D), and (E). At this point, the only remaining answer choice is (A), and it follows that response (A) is correct.

An alternate way to use the two templates is to say that the list of variables that could possibly appear fifth includes M from the first template, and N, S, and T from the second template. An examination of the answer choices reveals that only L in answer choice (A) does not appear as a possibility, and thus answer choice (A) must be correct.

Get the most out of your LSAT Prep Plus subscription.

Analyze and track your performance with our Testing and Analytics Package.